I am confused because the correct answer to this question seems like it's a necessary assumption and not a sufficient assumption, as the question stem would describe it to be. I generally regard necessary assumptions to be the "bare minimum" standard. It seems that "societies being geographically isolated enough not to have been influenced by any other society" only allows for, and does not guarantee, the conclusion to be drawn. When I really analyze the question I guess I could see how it's sufficient, but am I crazy to think that this reads so similarly to a necessary assumption? Is there a helpful technique for distinguishing the two in a situation like this?
Admin note: edited title
https://classic.7sage.com/lsat_explanations/lsat-60-section-1-question-22/
Maybe I still need to get used to the newer LR sections but I really am not a fan of the assumptions we are expected to make to get the right answer choices; they are analogous to those we have historically been penalized for making in other questions. (I'm referencing Q16 of this same section.) "Evolution does not always optimize the survival of an organism." How can we not only assert the existence of a group of people who think the opposite but also impose that view on them simply from that sentence? What if this were a necessary assumption or a most strongly supported/ MBT question with the same passage and they gave the following as an answer choice: "There exist some people who believe that evolution always optimizes the survival of an organism." No way would the LSAT make that the correct answer choice. We do not need to and simply are not logically allowed to make that leap from the information given. I understand different rules are prioritized for different question stems but the least the LSAT could do is be logically consistent. (Plus, I intentionally chose those analogy stems because they are governed by the same logical rules that (one would think) would hold just as much weight in the present question. For instance, I wouldn't have chosen an SA question because the same relevant logical rules do not apply as strongly there.) Does anyone have advice on how I can deal with making these inferential leaps going forward? Did I interpret this question incorrectly? Is there something I'm not realizing?